Sunteți pe pagina 1din 65

ELECTRICAL SYSTEM

1) Given the electrical figure below, solve for the transfer function of the system and then draw
the block diagram

Solution

KVL
Vi = Ri + L di/dt
Vo = L di/dt

Take the laplace transform

Vi(s) = I(s) [R + SL]


Vo(s) = s LI(s)

T(s) Vo/Vi = sL/R+SL

Vi-Vo/R Vo = sL/R+SL

I(s) = 1/R [Vi(s)- Vo(s)] Vo(s) = sLI(s)

1|Page
Block Diagram

2|Page
2) Given the electrical figure below, solve for the transfer function of the system and then draw
the block diagram.

Solution
KVL
Vi = Ri +L di/dt +Vo

Vi(s) = RI(s) + sLI(s) + Vo(s)

I(s) = Vi(s) – Vo(s)/R + SL

Vo = 1/C շ Idt

Vo(s) = (1/CS) I(s)

3|Page
3 Solve for the Vo of the transfer function of the system Using Nodal Analysis Method

L = dI/dt = VL = LS IL(s) = VL(s)


C = dVc/dt = Ic = Cs Vc(s) = Ic(s)
Vr = RIr = Vr(s) =R Ir(s)

Շ(d/dt f) =s F(s)
F(0) = 0 f(0) =! 0

@Node VA
VB = y
V(s)-VA/1Ω + 0-VA/1+1/2s + VB-VA/S = 0

4|Page
= -VA (-2s/2s+1) VA -1/S VA + 1/s VB = -U(s)
= (-1 – 1/S – (2s)/2s+1) VA + 1/S VB = -U(s)
= 2s^2 + s + 2s + 1 + 2s^2/2s^2 + s
= 4s^2 + 3s + 1/2s^2 + s
= 4s^2 + 3s + 1/(2s+s) VA – 1/s VB U(s) Equation 1

@Node VB
VA-VB/s + -VB/2 + -VB/2s + 1/s = 0
= 1/s VA + (-1/s – ½ - s/2s^2+1) VB = 0
= 1/s VA – (2s^3 + 6s^2 + s + 2/4s^3 + 2s) VB = 0 Equation 2

Eq 1 and Eq2
( ((4s^2+3s+1/(2s+1)(s)) ((2s^3+6s^2+s+2)/(4s^2+2) – 1/s ) VB = U(s)

U(s) = Y(s) = ( (4s^2+3s+1)(2s^3+6s^2+s+2) – (2s+1)(4s^2+2)/(2s+1) (s)(4s^2+2)

Y(s)/U(s) = ((2s+1)(4s^2+2))/((4s^2+3s+1)(2s^3+6s^2+s+2)-(2s+1)(4s^2+2))

Translational Mechanical System

5|Page
1. Find the equations describing the system.

Solution
Assume x2 > x1

ƒ1(t) – k1x1 + c(ẋ2 – ẋ1) + k2(x2 – x1) = m1ẍ1


ƒ2(t) – c(ẋ2 – ẋ1) – k2(x2 – x1) = m2ẍ2
Rearranging terms
m1ẍ1 + cẋ1 – cẋ2 + (k1 + k2)x1 – k2x2 = ƒ1(t)
m2ẍ2 – cẋ1 + cẋ2 – k2x1 + k2x2 = ƒ2(t)

6|Page
2. x1 and x2 denote the elongations of K1 and K2 respectively. Note that x1 is the displacement of
mass M1 with respect to a fixed reference but that x2 is the relative displacement of M2 with
respect to M1. When x1 = x2 = 0, all three spring shown in the figure are neither stretched nor
compressed. Draw the free-body diagram for each mass, including the effect of gravity and find
the differential equations describing the system’s behavior. Determine the values of x1 and x2 that
correspond to the static-equilibrium position, when ƒa(t) = 0 and when the masses are motionless.

Solution

If x1 and x2 are positive, then K1 and K2 are stretched and K3 is compressed. Under these
circumstances, K1 exerts an upward force on M1, and K2 and K3 exert downward forces on M1.
The relative velocity of M2 with respect to M1 is ẋ2 so frictional forces of Bẋ2 are exerted

7|Page
downward on M1 and upward on M2. The inertial force on M2 is proportional to its absolute
acceleration, which is ẍ1 + ẍ2. Summing the forces on each of the free-body diagrams gives
M1ẍ1 + K1x1 – Bẋ2 – (K2 + K3)x2 = M1g + ƒa(t)
M2ẍ1 + M2ẍ2 + Bẋ2 + (K2 + K3)x2 = M2g
To find the displacements x1 and x2 that correspond to the static-equilibrium position, we replace
ƒa(t) and all the displacement derivatives by zero. Then the equations reduces to
K1x1 – (K2 + K3)x2 = M1g
(K2 + K3)x2 = M2g
from which

If we want the differential equations in terms of displacements a1 and a2 measured with respect to
the equilibrium conditions given by third equations, then we can write x1 = x1 + a1 and x2 = x2 +
a2. Substituting these expressions into first equations and using second equations, we find that
M1ä1 + K1a1 – Bȧ2 – (K2 + K3)a2 = ƒa(t)
M2ä1 + M2ä2 + Bȧ2 + (K2 + K3)a2 = 0

8|Page
3. All springs are identical with constant K. Spring forces are zero when x1 = x2 = x3 = 0. Draw
FBDs and write equations of motion. Determine the constant elongation of each spring caused by
gravitational forces when the masses are stationary in a position of static equilibrium and when
ƒa(t) = 0.

Solution

9|Page
Summing the forces shown on each of the FBD and collecting terms.

Letting ƒa(t) = 0, replacing x1, x2, and x3 by the constant displacement x1o, x2o, and x3o, and
nothing that all the derivatives of these constant displacement are zero, we have the following
three algebraic equations.

Solving these equations simultaneously gives

The four spring elongations are x1o, x2o, and

Note that the elongations are not affected by the viscous damping coefficients B1 and B2.

10 | P a g e
Rotational Mechanical System

1. The system consists of a shaft of torsional stiffness K, two disks of polar moments of inertia J1
and J2, and a torsional damper B. Draw the necessary free-body diagrams and derive the
differential equations. Then express the equation in the second-order matrix form.

Solution

Assuming θ1 > θ2 > 0, Applying the moment equation about the mass centers along the
longitudinal axis,

The differential equations of the system are expressed in the standard input-output form as

And in the second-order matrix form as

11 | P a g e
2. In the system shown one flywheel (J1) is attached by a flexible shaft (Kr) to ground (the
unmoving wall) and has an applied torque, τa. A second flywheel (J2) is driven by friction
between the two flywheels (Br1). The second flywheel also has friction to the ground (Br2).
Derive equations of motion for the system shown.

Solution
First we must define our variables of motion. In this case there are two - the angles of the
flywheel. It is generally a good idea to define the variables in the same direction, so we
arbitrarily define them as positive in the counterclockwise direction.

Free body diagram at θ1 Free body diagram at θ2


There are 4 torques acting: There are 3 torques acting:
1. The external torque, τa, clockwise. 1. The torque due to Br2.
2. The torque due to Kr  If θ2 increases (counterclockwise),
 If θ1 increases the resulting torque is Br2·ω2,
(counterclockwise), Kr causes a clockwise.
clockwise torque on J1. 2. The torque due to Br1.
 The resulting torque is Krθ1,  If θ1 increases, the resulting torque
clockwise. on J2 is Br1·ω1 in the
3. The torque due to Br1. counterclockwise direction.
 If θ1 increases, the resulting  If θ2 increases, the resulting torque
torque on J1 is Br1·ω1 in the on J2 is Br1·ω2 in the clockwise
clockwise direction.

12 | P a g e

If θ2 increases, the resulting direction.
torque on J1 is Br1·ω2 in the  The torque due to Br1 is thus
counterclockwise direction. Br1·(ω2-ω1), clockwise (or Br1·(ω1-
 The torque due to Br1 is thus ω2) counterclockwise).
Br1·(ω1-ω2), clockwise.
4. The torque due to J1. 3. The torque due to J2.
 The resulting torque is  The resulting torque is
J1·α1 clockwise (the inertial J2·α2 clockwise.
torque is always in the opposite
direction from the define
positive direction).

3. An electric motor is attached to a load inertia through a flexible shaft as shown. Develop a
model and associated differential equations (in classical and state space forms) describing the
motion of the two disks J1 and J2.

Solution

The torsional stiffness of the shaft is given by:

13 | P a g e
And torque in the shaft by:

For disk 1:

or
For disk 2:

or
Classical form: Convert the two second-order equations into a single fourth-order equation.
Using the D-operator notation:
From the second equation:

Substitute into the first:

For the state-space form, let:

Substituting gives:

14 | P a g e
Systems with Gear
Ρ 3 ( s)
Problem No. 1. Find the transfer function for the following system,
T (s )

T(t) N1=6
J1=2

D1=18
N2=12 J2=55 N3 =5

�3(t)
N4=12 J3=9
K1=24

Solution:
2 2 2 2
N2 N4 N4
Jeq =J1(( )( )) ( )
N1 N3
+ J2
N3
+J3 Deq =D1
N4
( ) ( )
N3
=18
12
5
= 103.68 s

2
12 12 12 2
=2 (( )( )) ( )
6 5
+5
5
+9

Jeq = 83.88 s 2
Keq = 24

Ρ 3 (s )
=
(N2 N4
N1 N3 )
)( =
(126 )(125 )
=
4.8
T (s ) Jeq+Deq+Keq 83.88 s +103.68s+24 83.88 s 2 +103.68s+24
2

15 | P a g e
Ρ 3 ( s ) 4.8
= 2
T (s ) 83.88 s +103.68s+24

Ρ2 ( s )
Problem No. 2. Find the transfer function for the following system,
T (s )

T(t) N1
J1=30

�2(t) D1
J2=200
N2 N3

N4 D2
J3=200
K1

N1=10 N2=40
N3=25 N4=5
D1=300 D2=300
K1=10

Solution:
2 2 2 2
N2 N3 40 25
Jeq=J1 ( )
N1
+J2+J3
N4 ( ) ( )
=30
10
+200+200 ( )
5

Jeq=5680s2
2 2
N3 25
Deq =D1+D2 ( )
N4
=300+300 ( )
5
= 7800s

16 | P a g e
2
N2
Keq=K 1 ( )
N1
=160

Ρ2 ( s )
=
( N2
N1 )
= =
4( 4010 )
T ( s ) Jeq+Deq+Keq 5680 s +7800s+160 5680 s2 +7800s+160
2

Ρ2 ( s ) 4
=
T ( s ) 5680s 2 +7800s+160

Ρ3 ( s )
Problem No. 3. Find the transfer function for the following system,
T (s )
T(t) D1

J1 N1
K1
N2 N3
J2

�3(t)
N4
J3
K2

Where J1=2kg-m2, J2=1kg-m2, J3=16kg-m2, N1=12, N2=4, N3=8, N4=12, K1=15N-m/rad,


K2=20N-m/rad and D1=1N-m-s/rad.

Solution:
2 2
N2 N4 N4
Jeq=J1 (( )( )) ( )
N1 N3
+ J2
N3
+J3

2
4 12 12 2
(( )( )) ( )
=2
12 8
+1
8
+16 Jeq = 18.75s 2

17 | P a g e
2 2
Deq =D1 (( ( ) ( )( ))
N2 N4
)
N1 N3
) =1(
4
12
12
8
Deq=0.25s

2
N2 N4 4 12 2
Keq=K1
N1 (( )( ))
N3
+K2=15[ ( )( )
12 8
] +20 Keq=23.75

Ρ3 ( s )
=
( N2 N4
N1 N3 )
)( =
(124 )(128 )
=
0.5
T ( s ) Jeq+Deq+Keq 18.75s +0.25s+23.75 18.75 s2 +0.25s+23.75
2

Ρ3 ( s ) 0.5
=
T ( s ) 18.75 s2 +0.25s+23.75

Electromechanical System
Problem No. 1.
For the electromechanical system, find the transfer function, G(s) = �L(s)/Ea(s) given that
JL=25kg-m2, Ja = 1kg-m2, Da=7N-m-s/rad, DL = 35 n-m-s/rad and the torque speed curve of the
motor is Tm=-8�m+400 when the input voltage is 100V.
+ motor Ja Da N1=10
-
N2=20 N3=10

N4=25 �L(t) DL

Solution: JL
2 2
10 10
Jm=1+25
25(( )( ))20
Dm = Da + DL ( N3 N1
N4 N2 )
2
10 10
Jm=2 =7+35 ( 25 20 )
Dm = 8.4

Tm=-8�n + 400; Tstall= 400, �nl= 50

Kt TStall 400
= = = 4
Ra Ea 100

18 | P a g e
Ea 100
Kb= = = 2
ωnl 50

θm(s) Kt/(RaJm) 4/2 2


= = =
Ea 1 KtKb 1 s[s+8.2]
s[s+
Jm( Dm+
Ra )] s[s+ ( 8.4 ) + ( 4 )( 2 ) ]
2
10 10
(
θL ( s ) = θm ( s ) )
25 20
1
θL ( s ) =θm ( s )
5
1
θL(s)
=
()
2
5
Ea s[s+8.2]

θL(s) 0.4
=
Ea s[s+8.2]
Probelm No.2.

Find the transfer function, G(s)= �L(s)/Ea(s), for the following electromechanical system,

Where J1=10kg-m2, N1=20, B1=5N-m-s/rad, N2=30, B2=10N-m-s/rad, J2=15kg-m2 and the


torque-speed curve is shown below:
Tm

100N-m
50 V

300 rad/s
Solution:
Jm = J1+ J2 (N1/N2)2 Bm = B1 + B2 (N1/N2)2 TStall =100

19 | P a g e
= 10+ 15 (20/30)2 = 5+ 10 (20/30)2 �No Load = 300
Jm = 16.6667 Bm = 9.4444 ea= 50
Kt Tstall 100
= = = 2
Ra ea 50
ea 50 1
Kb= = =
ωnl 300 6
2
θm(s) Kt/(RaJm) (16.6667 ) 0.1120
= = =
Ea 1 KtKb s[s+0.5867]
s[s+
Jm ( Bm+
Ra )
] s[s+ 1
16.6667 (
9.4444+2
1
6 ( )) ]
20
θL ( s ) = θm ( s ) ( )
30

θL(s)
=
0.1120 ( 2030 )
=
0.0747
Ea s[s+0.5867] s [ s+0.5867 ]
Problem No. 3.
For the electromechanical system, find the transfer function, G(s) = �L(s)/Ea(s)
+

�m(t) N1
Ea(t) Motor

N2 N3

�L(t)
N4 DL

JL

Consider ea=50 DC, N1=20, N2=80, N3=5, N4=20, JL=200kg-m2, Dl=400 N-m-s/rad.
The torque speed curve for the motor is:

20 | P a g e
Tm Tstall=200
�No Load=25

200 50 ea=50


25
Solution:
2 2
N1 N3 20 5
Jm=JL( ( )( )
N2 N4
= 200 (( )( ))
80 20

25
Jm =
32
2 2
N1 N3 20 5
Dm=DL
N2 (( )( )) N4
= 400(( )( ))
80 20

25
Dm =
16

Kt Tstall 200
= = = 4
Ra Ea 50
ea 50
Kb= = = 2
ωnl 25
25
4/( )
θm(s) Kt/(RaJm) 32
= =
Ea 1 KtKb 1 25
s[s+
Jm
Bm+
Ra (
] s[s+
25 16
( )
)
+4(2) ] ( )
32
θm(s) 5.12
=
Ea s(s+12.24)

θL ( s ) = θm ( s ) (2080 )(520 )
θL(s)
=
5.12
=
( 116 )
0.32
Ea s(s+12.24) s(s+12.24)

21 | P a g e
State Space Representation
Problem 1:
The state vector, written in the Laplace domain,
X (s) =[sI − A]-1 BU(s) (i)
from the previous example is:
-1
Vc(s) IL(s) = s −1/C 0 Vin(s).
1/L s + R/L 1/L (ii)
The determinant of [sI − A] is
det [sI − A] = (s2 + (R/L)s + (1/LC)) , (iii)
and the adjoint of [sI − A] is
adj s −1/C = s + R/L 1/C (iv)
1/L s + R/L -1/L s
From Example .5 and the previous example, the output equation vL(t) = −vC − RiL + Vs(t)
specifies that C = [−1 − R] and D = [1]. The transfer function, Eq. (26) is:

H(s) = (v)
Since
C adj (sI − A) B = [−1 –R] s + R/L 1/C 0 = (vi)
−1/L s 1/L
the transfer function is

H(s) = , (vii)

Problem 2:
Consider the following state equations:
x˙1 = x2(t) ,
x˙ 2 = x3(t) ,
x˙ 3 = − 6x1(t) − 11x2(t) − 6x3(t) + 6u(t) ,

22 | P a g e
y = x1(t) ,
and determine the controllable canonical form.
Solution: we write the following high order differential equation:
y(t) + 6 y(t) + 11 y(t) + 6y(t) = 6u(t) .
The state variables x(t) = y, x2(t) = ˙y, and x3 = ¨y. Hence, we get
x˙1 =x2(t) ,
x˙2 =x3(t) ,
x˙3 = y(t) = −6¨y(t) − 11 ˙y(t) − 6y(t) + 6u(t)
= − 6x3(t) − 11x2(t) − 6x1(t) + 6u(t) .
In matrix form, we have
x˙(t) = 0 1 0 0
0 0 1 x(t) + 0 u(t),
-6 -11 -6 6
y (t) = [ 1 0 0 ] x(t)

Problem 3:
Find the State equations for the series R-L-C electric circuit shown

23 | P a g e
capacitor voltage VC(t) and the inductor current IL(t) are state variables
vc = 0 1/C vc + 0 vin
iL -1/L -R/L iL 1/L

y(t) = [ 1 0 ] vc + [ 0 ] Vin
iL
Prove: Appling KVL on the circuit
Vs(t) = R* i + vc + L… (1)
The relation of capacitor voltage and current i = c
Then
x = = = x2
from equation (1)
x2 = = -vc – R * i + vs(t)
x2 = [ -x1 – R * x2 + u(t)+]
y = v c = x1

24 | P a g e
Transfer Function to State Space ( Single Polynomial )
1. Find the state-space representation of the following transfer function system.

R(s) C(s)
160
G ( s )= 3 2
s + 10 s +31 s +20

Solution:

s 3 +10 s 2+ 31 s+20 =
⃛x + 10 x́ +31 x́ +20 x = r
⃛x =−10 x́−31 x́ – 20 x + r
x 3=−10 x 3−31 x 2−20 x 1+r

x 1=x x́ 1=x 2

x 2=x́ x́ 2=x 3

x 3=x́ x́ 3=x 4

x 4= ⃛x

0 1 0 x1 0
X= 0 0 1 x2 + 0 r
−20 −31 −10 x3 1

x1
Y= 160 0 0 x2
x3

25 | P a g e
2. Find the state-space representation of the following transfer function system.

R(s) C (s)
210
G ( s )= 4 3 2
s +6 s +16 s +20 s +30

Solution:

s 4 +6 s3 +16 s2 +20 s+30 =


⃛x́ + 6 ⃛x +16 x́ +20 x́+ 30 x=r
⃛x́ =−6 ⃛x −16 x́−20 x́−30 x+ r

x 4=−6 x 4 −16 x 3 −20 x 2−30 x 1 +r

x 1=x x́ 1=x 2

x 2=x́ x́ 2=x 3

x 3=x́ x́ 3=x 4

x 4= ⃛x x́ 4=x 5

x 5= ⃛x́

0 1 00 x1 0
0 0 10 x2 0
X= + r
0 0 0 1 x3 0
−30 −20 −16 −6 x4 1

x1
x2
Y= 210 0 0 0
x3
x4

26 | P a g e
3. Find the state-space representation of the following transfer function system.

R(s) C (s)
16
G ( s )= 5
s + 21 s + 19 s 3+ 6 s 2+2 s+1
4

Solution:

s 5 +21 s 4 +19 s 3+6 s 2+ 2 s+1 =


⃛x́ + 21⃛´ x +19 ⃛x +6 x́ +2 x́+1 x=r
⃛x́ =−21 ⃛x́ −19 x⃛ −6 x́−2 x́−1 x+ r

x 5=−21 x 5−19 x 4−6 x 3 −2 x 2−1 x 1+r

x 1=x x́ 1=x 2

x 2=x́ x́ 2=x 3

x 3=x́ x́ 3=x 4

x 4= ⃛x x́ 4=x 5

x 5= ⃛x́ x́ 5=x 6

x 6= ⃛x́

0 1 0 0 0 x1 0 Y = 16 0 0 0 0 x1

0 0 1 0 0 x2 0 x2

X= 0 0 0 1 0 x3 + 0 x3

0 0 0 0 1 x4 0 x4

-1 -2 -6 -19 -21 x5 0 x5

27 | P a g e
Transfer Function to State Space ( Multiple Polynomial )

1. Find the state-space representation of the following transfer function system.

C (s) C (s)s 2+7 s+2 R (s)


=G s = 3
( )
R (s ) s + 9 s 2+ 26 s+ 24

Solution:
s
s
(¿¿ 2+7 s +2)R(s)
(¿ ¿ 3+9 s2 +26 s+24 )C ( s ) =¿
¿
⃛x + 9 x́+26 x́ +24 x = r
⃛x =−9 x́−26 x́ – 24 x + r
x 3=−109−26 x 2−24 x 1 +r

x 1=x x́ 1=x 2

x 2=x́ x́ 2=x 3

x 3=x́ x́ 3=x 4

x 4= ⃛x

0 1 0 x1 0
X= 0 0 1 x2 + 0
−24 −26 −9 x3 1

28 | P a g e
x1
Y = 2 7 1 x2
x3

2. Find the state-space representation of the following transfer function system.

C (s) R (s)
C (s) 5 (s +4 )
=G ( s )= 3
R (s ) s + 22 s2 +16 s+50

Solution:
s
2
(¿ ¿ 3+22 s +16 s +50 )C ( s )=5( s+ 4)R (s )
¿
⃛x + 22 x́ +16 x́ +50 x = r
⃛x =−22 x́−16 x́ – 50 x + r
x 3=−22 x 3−16 x 2−50 x 1+ r

x 1=x x́ 1=x 2

x 2=x́ x́ 2=x 3

x 3=x́ x́ 3=x 4

x 4= ⃛x

0 1 0 x1 0
X= 0 0 1 x2 + 0
−50 −16 −22 x3 1

x1
Y = 20 5 x2
x3

29 | P a g e
3. Find the state-space representation of the following transfer function system.

R (s) 2 C (s)
s +3 s+3
G ( s )= 3 2
s + 2 s +11 s+6

Solution:

(s 3 +2 s 2+11 s+ 6)C ( s )=s 2 +3 s +3 R (s )


⃛x + 2 x́ +11 x́ +6 x = r
⃛x =−2 x́−11 x́ –6 x + r
x 3=−2 x 3−11 x 2−6 x 1 +r

x 1=x x́ 1=x 2

x 2=x́ x́ 2=x 3

x 3=x́ x́ 3=x 4

x 4= ⃛x

30 | P a g e
0 1 0 x1 0
X= 0 0 1 x2 + 0
−6 −11 −2 x3 1

x1
Y = 3 3 1 x2
x3

Stability
Problem 1:
Find stability of the following system given by

and H(s) = 1 using Routh-Hurwitz stability criterion.


Solution:

In the system,
Method-I,
Characteristics equation, B(s) = s2 + s + K =0

31 | P a g e
The system is always stable for K>0.
Method-II,
Characteristics equation, B(s) = s2 + s + K = 0
Here, Routh array is

There are no sign changes in first column elements of this array.Therefore, the system is always
stable for K>0.

Problem 2:
Find stability of the following system given by B(s) = s2 + 2s2 + 10 using Routh-Hurwitz
stability criterion.
Solution: In this problem, given characteristics equation is
B(s) = s2 + 2s2 + 10 and
Routh’s array is

There are two sign changes in first column elements of this array. Therefore, the system is
unstable.
Problem 3:
Examine stability of the following system given by s4 + 5s3+ 2s2 + 3s + 1 = 0 using Routh-
Hurwitz stability criterion. Find the number of roots in the right half of the s-plane.
Solution: In this problem,
Routh’s array is

32 | P a g e
There are two sign changes in first column elements of this array. Therefore, the system is
unstable. There are two poles in the right half of the s-plane.

Problem 4:
Find stability of the following system given by

and H(s) = 1 using Routh-Hurwitz stability criterion.


Solution:
In the system,

Method-I,
General form of characteristics equation, B(s) = a3s3 + a2s2 + a0 = 0
And in this system, characteristics equation is B(s) = s3 + 6s2 + 8s + K = 0
Here,sufficient condition of stability suggests

Therefore, the system is always stable for K < 48


33 | P a g e
Method-II,
Characteristics equation is B(s) = s3 + 6s2 + 8s + K = 0
andRouth’s array

There are no sign changes in first column elements of this array if K < 48. Therefore, the system
is always stable for 0 < k < 48.

Marginal Stability

1. Given the system below find the range of the gain K that will lead to stability, unstability
and marginal stability.

Closing the loop,

34 | P a g e
1
• If K is positive (assumed) all the elements in the first column are positive except that the s
entry may be positive/nagative.
• If K < 1386, the system will remain stable.
• If K > 1386, the system will remain unstable.
• If K = 1386, a row of zeros, so using
2
P(s)=18 s +1386
dP(s )
=36 s+0
ds

35 | P a g e
= Since there are no sign change from s 2 to s 0 the even polynomial has no unstable roots, only
jω roots, therefore the system is marginally stable when K = 1386.

K
G(s )=
2. For the unity feedback system with [(s 2 +2 s +2)( s+2 )] find the range for
stability, instability and the value of gain for marginal stability.

Solution. since P=0, we want no encirclements of -1 for stability.


Hence. A gain less than unity at +/- 180 is required. first set K=1 and sketch the portion of
Nyquist diagram along the positive imaginary axis as shown in (a). We find the intersection with
the negative real axis as Setting the imaginary part = zero, we find and substituting yields real
part –(1/20) =
This closed loop system is stable if the magnitude of the frequency response is less than unity at
180o. Hence, the system is stable foe K<20, unstable for K>20, and marginally stable for K=20
(frequency at marginal stability is )

3.

36 | P a g e
3 2
Since all the coefficients of the closed-loop characteristic equation a s +10 s +31 s+1030 are
present, the system passes the Hurwitz test. So we must construct the Routh array in order to test
the stability further.

1
1
−72
-it is clear that column 1 of the Routh array is: 103
-it has two sign changes ( from 1 to -72 and from -72 to 103). Hence the system is marginally
stable.

STABLE
If all the poles of the system lie in leftjhalf
 plane the system is
said to be Stable.

LHP RHP
1.

X
-3
37 | P a g e

s-plane
C=10
A=1
B=3
Zeroes:0
Poles: -3

2. Poles: -3, -6

The -1+j0 point is not encircled so


N=0. There are no poles of L(s) in the right
half plane so P=0. Since N=Z-P, Z=0. This
means that the characteristic equation of the
closed loop transfer function has no zeros in
the right half plane (the closed loop transfer
function has no poles there). The system is
stable.

3.

Because there is a pole at the origin, we can infer that the


counterclockwise 180° detour around the origin in "s" yields a clockwise 180° detour in "L(s)" that is not
shown in the Matlab plot. Hence the -1+j0 point is not encircled and the system is stable.

38 | P a g e
UNSTABLE

1.

39 | P a g e
The Matlab plot is initially quite hard to decipher,
But it becomes clear if we zoom in (and display the stability margins, which are both negative,
indicating instability). We can check this by finding the location of the zeros of the characteristic
equation: This has roots at s=-5.36, 1.18±4.15j so the system is unstable as expected.

2.

where a∈Ca∈C is the system pole. If we observe the behavior of the system in time we have

Since a is complex we can write it as a=b+jc where b is the real part of a and c the imaginary
part. Then the system becomes:

Note that will cause the system to oscillate, while will determine how (and if) the x will
converge to u.
If b<0 the system will go to zero since →0 when t→∞. Meanwhile, if b>0 the system will
diverge since →∞ when t→∞.
Note that c does not play a role here. So independently of the imaginary part, the real part of the
pole needs to be negative for stability.

For the case b=0 the system will neither converge nor diverge, however stability is defined by
strict convergence, so b=0 is not stable

3.

40 | P a g e
The -1+j0 point is encircled one in the clockwise direction
so N=1. There is one pole of L(s) in the right half plane so
P=-1. Since N=Z-P, Z=2. This means that the
characteristic equation of the closed loop transfer function
has two zeros in the right half plane (the closed loop
transfer function has two poles there). The system is
unstable.
This has roots at s=-1.53, 1.26±7.95j so the system is
unstable as expected.

FIRST ORDER(POLES AND ZEROS)


1.

41 | P a g e
In a series RCL circuit driven by a constant emf, the natural response of the circuit is given by

dt2d2i+4dtdi+4i=0

for which the initial conditions are i(0) = 2 A and

di/dt at t=0 is 4

Solution:

L=1, R=4, 1/C=4 so C=1/4

R^2=16

4L/C = 4/ 1/4 =16

So R^2 = 4L/C and therefore we have critical damping

2.

42 | P a g e
Find y(t) from the given figure

3. The standard form of a second-order transfer function denominator is

43 | P a g e
s 2 + 2ζωns + ω 2 n

By equating coefficients and solving for damping ratio ζ and (undamped) natural
frequency ωn, we get:

(a) (s + 3)(s + 6) = s 2 + 9s + 18 from which we find ωn = √ 18 = 4.24 rad/s, ζ = 1.06


(overdamped)

(b) s 2 + 6s + 144 from which we find ωn = √ 144 = 12 rad/s, ζ = 0.25 (underdamped)

Poles and Zeros (2nd Order)

44 | P a g e
1. A second-order system has a pair of complex conjugate poles a s = −2±j3 and a single zero
at the origin of the s-plane. Find the transfer function and use the pole-zero plot to
evaluate the transfer function at s =0+ j5.

The pole-zero plot for a second order system with a zero at the origin and

T
2. Which of T 2 and 3 is a better approximation to T 1
3.

45 | P a g e
T
The step responses for T 2 and 3 have the following form:
pt −2 t
c i ( t )=1+ k 1 e i + k 2 e cos ( 4 . 352t +φ )

Solution: The pole for T 2 is 5 times the real component of the dominant poles and can
therefore be approximately neglected.

4. (Overdamped Response)

The output can be calculated as

9 K1 K2 K3
C( s )= = + +
s (s 2 + 9 s+ 9) s s+ 7. 854 s+1 .146

46 | P a g e
The output, even without calculations can be written as,

−7 . 854t −1. 146t


c (t )=K 1 + K 2 e +K 3 e

DAMPED RESPONSE

47 | P a g e
1. A system has a pair of complex conjugate poles p1, p2 = −1 ± j2, a single real zero z1 =
−4, and a gain factor K = 3. Find the differential equation representing the system.

The transfer function is


H(s) = K s − z/(s − p1)(s − p2)
= 3 s − (−4)/(s − (−1 + j2))(s − (−1 − j2))
= 3 (s + 4)/ s2 + 2s + 5
and the differential equation is
d2y/dt2 + 2dy/dt + 5y
= 3 du/dt + 12u

2. Find the pole-zero representation of the system with the transfer function:

First rewrite in our standard form

48 | P a g e
So the pole-zero representation consists of:

1. a constant term, k=3,

2. zeros at s=-1 and s=-2, and

3. polese at s=-1+j, s=-1-j and s=-3.

The plot below shows the poles (marked as "x") and the zeros (marked as "o") of the
response. The gain, k, is not shown.

3. Find the pole – zero plot of the system represented by the transfer function

H(z)= z^−2+z^−3 / 1−3,6z^−1 + 4,59z ^-2 − 2,38z^−3 + 0,39z^−4

49 | P a g e
Multiplying both numerator and denominator by Z^4 we get

H(z)= z2+z / z4−3,6z3 + 4,59z2 − 2,38z + 0,39


=N(z) / D(z)

Factorize out the numerator and denominator :

N(z)=z(z+1)
D(z)=(z−1)^2 (z^2 − 1.6z + 0.39)

Thus the zeros of the system are

z(z+1) =0 ⇒ z = 0, z = −1
and the poles are

(z−1)^2 (z2−1.6z+0.39)=0
z=1(double)
z=0.8+j0.5
z=0.8−j0.5

the pole – zero plot of the system .

50 | P a g e
Multiple Input Multiple Output

1. Blending System

Solution
Y1 = WA
Y2 = WB
Y1 = W
Y2 = X
Y= n output
G= nxm TF format
U= input
Y = Gu

Y1 G11
U1 G12
Y2 G21
U2 G22

2 INPUT AND 2 OUTPUT

51 | P a g e
Final Diagram

2) Find C1/R1 from block diagram

52 | P a g e
C1/R1 ; Assume R2 = 0 & C2 = 0
Step 1

Step 2

53 | P a g e
Step 3

Step 3
G1 and feedback
(G1)/1-G1(H1H2G4G5/1+G4)
= 1+ G4 - (G1) (G4) (G5) (H1) (H2))

54 | P a g e
Final

3.Find C1/R1 AND C2/R2 from block diagram

55 | P a g e
R2 = 0
C1 is output

C1(s)/R1(s) = (G1/1 + [ G2 (-G4) G3 G1])


= (G1/1 - G1 G2 G3 G4)

R1 = 0
C2 = OUTPUT

C2(S)/(R1(S) = [-G4/1 + G1 G2 G3 - G4]

=-G4/1 - G1 G2 G3 G4

56 | P a g e
Multiple Input Single Output
1.

57 | P a g e
2.

58 | P a g e
3.

59 | P a g e
Single Input Single Output

1. USING THE BLOCK DIAGRAM ALGEBRA. FIND THE TRANSFER FUNCTION y/r.

60 | P a g e
1. USING THE BLOCK
DIAGRAM ALGEBRA. FIND
THE TRANSFER FUNCTION y/r.

61 | P a g e
62 | P a g e
2. USING THE BLOCK DIAGRAM ALGEBRA. FIND THE TRANSFER FUNCTION y/r.

Step 1 − Use Rule 1 for blocks G1G1 and G2G2. Use Rule 2 for blocks G3G3 and G4G4. The
modified block diagram is shown in the following figure.

Step 2 − Use Rule 3 for blocks G1G2G1G2 and H1H1. Use Rule 4 for shifting take-off point
after the block G5G5. The modified block diagram is shown in the following figure.

63 | P a g e
Step 3 − Use Rule 1 for blocks (G3+G4)(G3+G4) and G5G5. The modified block diagram is
shown in the following figure.

Step4 –Use Rule 3 for blocks (G3+G4)G5(G3+G4)G5 and H3H3. The modified block diagram

is shown in the following figure.

Step 5 − Use Rule 1 for blocks connected in series. The modified block diagram is shown in the

following figure.

64 | P a g e
Step 6 − Use Rule 3 for blocks connected in feedback loop. The modified block diagram is

shown in the following figure. This is the simplified block diagram.

Therefore, the transfer function of the system is

65 | P a g e

S-ar putea să vă placă și